Question

please help me!! will comment and rate Thank you!!

The figure below shows a top view of a bar that can slide on two frictionless rails. The resistor is R 7.000, and a 2.50-T ma

0 0
Add a comment Improve this question Transcribed image text
Answer #1

(ay Papp irduced emp e BIV BIV R Curent 1 n ded r ilB mognelk ferce due ho fapid 12 20.7 N rate erug Power deliveted-Fv 20.7

Add a comment
Know the answer?
Add Answer to:
please help me!! will comment and rate Thank you!! The figure below shows a top view...
Your Answer:

Post as a guest

Your Name:

What's your source?

Earn Coins

Coins can be redeemed for fabulous gifts.

Not the answer you're looking for? Ask your own homework help question. Our experts will answer your question WITHIN MINUTES for Free.
Similar Homework Help Questions
  • The figure below shows a top view of a bar that can slide on two frictionless...

    The figure below shows a top view of a bar that can slide on two frictionless rails. The resistor is R - 6.80 O, and a 2.50-T magnetic field is directed perpendicularly downward, into the page. Let ! - 1.20 m. XX XX R (a) Calculate the applied force required to move the bar to the right at a constant speed of 1.90 m/s. N (to the right) (6) At what rate is energy delivered to the resistor? w

  • The figure below shows a top view of a bar that can slide on two frictionless...

    The figure below shows a top view of a bar that can slide on two frictionless rails. The resistor is R = 5.00 Ω, and a 2.50-T magnetic field is directed perpendicularly downward, into the page. Let ℓ = 1.20 m. A vertical bar and two parallel horizontal rails lie in the plane of the page, in a region of uniform magnetic field, vector Bin, pointing into the page. The parallel rails run from left to right, with one a...

  • 4. -/10 points My Notes Ask Your Teacher The figure below shows a top view of a bar that can slide on two frictionl...

    4. -/10 points My Notes Ask Your Teacher The figure below shows a top view of a bar that can slide on two frictionless rails. The resistor is R -6,40 0, and a 2.50-T magnetic field is directed perpendicularly downward, into the page. Let l = 1.20 m. XXX Bin x x xxx (a) Calculate the applied force required to move the bar to the right at a constant speed of 1.50 m/s. N (to the right) C (b) At...

  • The figure below shows a top view of a bar that can slide on two frictionless...

    The figure below shows a top view of a bar that can slide on two frictionless rails. The resistor is R = 6.40 s, and a 2.50-T magnetic field is directed perpendicularly downward, into the page. Let l = 1.20 m. X X X X X X Bin X X X X X x X x X X X R X X X X X X *F app X X X X X X X X X (a) Calculate the...

  • The adjacent figure shows a top view of a bar that can slide on two frictionless...

    The adjacent figure shows a top view of a bar that can slide on two frictionless rails. The resistance is R = 6.2. A 2.5 T magnetic field is directed perpendicularly downward into the paper. Let l = 1.2 m. (a) Calculate the applied force required to move the bar to the right at a constant speed of 2 m/s. X xx xx x x Bin (b) At what rate is energy delivered to the resistor? app

  • QUESTION 12 2, and a 2.50-T magnetic field is directed perpendicularly downward, The figure below shows...

    QUESTION 12 2, and a 2.50-T magnetic field is directed perpendicularly downward, The figure below shows a top view of a bar that can slide on two frictionless rails. The resistor is R = 6.20 into the page. Let l = 1.20 m. x * X * * * BAM * * * * * * * * X X X * * * * * * X X X * * * * * R . * * X...

  • Question 31.8 and question 31.27 000 W A strong a unifrm magnetic field of 1.60 T...

    Question 31.8 and question 31.27 000 W A strong a unifrm magnetic field of 1.60 T over a cross-sectional area of 0320 m2. A col' having 230 turns and a total resistance of 19.0 n is placed around the electromagnet. The current in the electromagnet is then smoothly reduced until it reaches zero in 20.0 ms. What is the current induced in the col? 31 P 027 MI F The figure below shows a top view of a bar that...

  • The figure below shows a bar of mass m = 0.280 kg that can slide without...

    The figure below shows a bar of mass m = 0.280 kg that can slide without friction on a pair of rails separated by a distance ℓ = 1.20 m and located on an inclined plane that makes an angle θ = 29.5° with respect to the ground. The resistance of the resistor is R = 2.20 Ω, and a uniform magnetic field of magnitude B = 0.500 T is directed downward, perpendicular to the ground, over the entire region...

  • Please help on physics question An aluminum bar is moved across conducting rails as shown below....

    Please help on physics question An aluminum bar is moved across conducting rails as shown below. A constant external magnetic field is directed into the page. The length of the bar is 1.20 m. The magnitude of the external magnetic field is 2.50 T. The resistance R is 6.00 Ohm. At what speed (in m/s) should the bar be moved to produce a current of 0.500 A in the resistor?

  • A pair of conducting, parallel, frictionless rails is mounted on an insulating platform. The distance between...

    A pair of conducting, parallel, frictionless rails is mounted on an insulating platform. The distance between the rails is L = 0.20 m. The rails are connected on one end by a R = 10.12 resistor. A conducting bar of mass 1.2 kg can slide on the rails without friction. When the conducting bar is at x = 0, the enclosed area of the loop is 0.03 m2. There is zero resistance in the conducting bar or rails. A uniform...

ADVERTISEMENT
Free Homework Help App
Download From Google Play
Scan Your Homework
to Get Instant Free Answers
Need Online Homework Help?
Ask a Question
Get Answers For Free
Most questions answered within 3 hours.
ADVERTISEMENT
ADVERTISEMENT
ADVERTISEMENT